Substantial economic growth must be preceded by technological innovations that expanding industries incorporate into ...

on December 24, 2019

A and D

How is D the correct answer? This is how I diagrammed: Premise: If technological innovations then economic growth Premise: Technological innovations exist Conclusion: Economic boom (or growth). The passage says that economic growth is preceded technological innovations...to precede something is to come before...doesn't that mean technological innovation is the sufficient condition?

Reply
Create a free account to read and take part in forum discussions.

Already have an account? log in

Shunhe on January 1, 2020

Hi @tomgbean,

Unfortunately, your diagramming isn't quite right, which made you eliminate (D) wrongly. I'll diagram it first and then go through (A) and (D). Note that the first sentence reads, "substantial economic growth MUST BE preceded by technological innovations..." The "must" tips us off to the presence of a conditional. Personally, I don't think of it in terms of necessary/sufficient conditions, which can be a bit confusing. Instead, I think of it this way. If economic growth must be preceded by technological innovations, that means that if we have economic growth, then we had technological innovations as well. Otherwise, we wouldn't have the economic growth. And "if X, then Y" is easily translatable into "X - > Y." So here, we would have:

Substantial economic growth - > Preceded by technological innovations

For the second sentence:

Worldwide ban on fossil fuels - > Technological innovations

And from that, we conclude:

Worldwide ban on fossil fuels - > Economic boom

Let's talk about (A) first. Does the argument assume the truth of the conclusion for which it purports to be providing evidence, essentially making a circular argument? No, as the conclusion is that a ban on fossil fuels would be followed by an economic boom, but that isn't one of the two premises we identified. Thus, we can rule out (A).

We can see, however, that (D) is correct after we've diagrammed it correctly. The argument confuses a necessary condition with a sufficient condition. Note that the author wants to conclude the following:

Ban - > Technological innovations - > Economic growth

But because the first sentence of the stimulus actually tells us

Economic growth - > Technological innovations

We can't actually make the logical chain the author fallaciously makes. The author makes this mistake because he believes Technological innovations - > Economic Growth, and (D) is therefore the correct answer. Hope this helps, and feel free to ask any questions of clarification.